Test Your Knowledge: Expert Review and Answer Explanations

You might also like

Download as pdf or txt
Download as pdf or txt
You are on page 1of 52

Test Your Knowledge: Expert Review and Answer Explanations

Complete the quiz for each module and then watch the video review.

Module 1 Question 1

A young associate works for a small plaintiffs' personal injury firm. One
afternoon, she is driving from a deposition back to the office when she
encounters a sudden stop in traffic. She calls her supervising partner to
advise that she will be late getting back to the office. While the two
are talking on the phone, the young associate notices that the cause for
the traffic jam is a four-car accident up ahead. She sees several injured
people on the side of the road and mentions this to the partner. The
partner tells the associate to pull over and give business cards to the
injured people. Knowing this is an ethical violation, the associate says
she does not feel comfortable doing so. The partner responds, "I'm making
a judgment call here. Go ahead and hand out your business card. If you
get in trouble for this, I'll handle it." The young associate does indeed
hand out business cards to the parties involved in accident. Assume that
the court finds that this is an ethical violation.

Will the young associate be subject to discipline for her actions?

(A) No, because the young associate was following the orders of the
supervisory attorney.

(B) No, because the young associate acted in accordance with a supervisory
lawyer's reasonable resolution of an arguable question of professional
duty.

(C) Yes, because following a supervisor's orders is never a defense to an


ethical violation of a subordinate lawyer.

(D) Yes, because the young associate knew it was a violation to hand out
business cards at the scene of the accident.
Module 1 Question 2

Two attorneys were the sole partners in a law firm. Their office manager
was not a lawyer. The office manager had three friends who were seriously
injured in an automobile accident. The office manager encouraged his three
friends to hire the law firm to represent the friends in a lawsuit to re-
cover damages for the friends' injuries. The two attorneys were able to
negotiate a multimillion dollar settlement for the office manager's
friends, which resulted in substantial attorney fees for the firm. The two
attorneys were grateful to the office manager for referring his friends to
the firm. Under the law firm's profit-sharing plan, the office manager re-
ceived one percent of the attorney fee that the two attorneys received for
representing the office manager's friends.

Is the payment to the office manager proper?

(A) Yes, because the office manager referred the clients to the two attor-
neys.

(B) Yes, because the payment is pursuant to the firm's profit-sharing


plan.

(C) No, because the office manager is not a lawyer.

(D) No, because the office manager solicited his friends to hire the two
attorneys.
Module 2 Question 1

An attorney practices business law and regularly incorporates small busi-


nesses. Several years ago, the attorney incorporated a gift shop for a
client. She has not heard from that client since the incorporation was
completed. The attorney learns of new business regulations that she be-
lieves would affect the gift shop owner's business.

Would the attorney be subject to discipline if she calls the gift shop
owner to advise her of the new regulations?

(A) No. In fact, the attorney would be subject to discipline if she did
not contact the gift shop owner about the new regulations.

(B) No, because the gift shop owner has a prior professional relationship
with the attorney.

(C) Yes, because the gift shop owner is no longer the attorney's client.

(D) Yes, although the attorney would be permitted to send the gift shop
owner a letter regarding the new regulations.

Module 2 Question 2

A client asks his attorney to negotiate a sales contract. The client is


pressed for time and offers the attorney twice her normal rate to expedite
the request. The attorney is very busy but thinks the client will pay even
more to compensate her. Over the phone, the attorney explains her workload
predicament to her client and makes a counter-offer to negotiate the sales
contract for a percentage of the sales price. The client agrees.

For which of the following could the attorney be subject to discipline?

(A) Failing to put the fee agreement in writing.

(B) Profiting excessively from a time-disadvantaged client.

(C) Accepting the employment, because doing so would preclude other em-
ployment by the attorney.

(D) The fee, which is unreasonably high for this service.


Module 3 Question 1

An electrician and a lawyer went to high school together. One afternoon,


the electrician comes to the lawyer's office. He sits down, pulls a gun
out of his pocket, and tells the lawyer, "I shot my wife this morning."
The lawyer quickly tells the electrician to put the gun back in his pocket
and to say nothing more. He then tells the electrician that his practice
is limited to tax and real estate matters, that he is not qualified to
represent the electrician or give competent legal advice, and that the
electrician should seek representation from a good criminal defense attor-
ney. The lawyer hands the electrician a list of competent criminal defense
attorneys and asks him to leave. The electrician is arrested and charged
with murder. The lawyer is arrested for evidence tampering after the elec-
trician tells the police that the lawyer told him to hide the gun.

Would the lawyer be subject to discipline if he reveals that he only told


the electrician to put the gun back in his pocket?

(A) Yes, because the lawyer's duty of confidentiality prohibits him from
disclosing his conversations with the electrician.

(B) Yes, if the disclosure would be detrimental to the electrician's in-


terests.

(C) No, if the disclosure is necessary to defend against the evidence-tam-


pering charge.

(D) No, because the lawyer declined to represent the electrician, so the
attorney-client privilege does not apply.

Module 3 Question 2

A business owner has been sued for violations of the Americans With Disa-
bilities Act. He calls his friend, who is a lawyer, to ask if the friend
could represent him in the case. In the course of the conversation, he
confided in his lawyer friend about an unrelated legal matter, in which he
was represented by another attorney. Because the lawyer friend had no ex-
pertise or experience with ADA cases, he declined the representation. Sub-
sequently, the lawyer friend is called as a witness in the unrelated legal
matter and agrees to disclose the substance of his conversation with the
business owner over the business owner's objection.

Is the lawyer friend's action proper?

(A) Yes, because the lawyer friend could not competently represent him in
the Americans With Disabilities Act case.

(B) Yes, but only the portion of the conversation regarding the legal mat-
ter unrelated to the initial consultation.

(C) No, because the entire conversation is confidential.

(D) No, unless the lawyer believes disclosure is in the business owner's
interest.
Module 4 Question 1

A defendant is charged with grand larceny. She tells an attorney that she
is innocent and asks the attorney to defend her. As it turns out, the at-
torney's sister is handling the defendant's prosecution. The attorney dis-
closes to the defendant that her sister is the prosecutor and then asks to
hear the defendant's side of the story. The attorney agrees to represent
the defendant. During the course of the prosecution, the attorney does not
talk to her sister about the case or anything else.

Is the attorney subject to discipline for a conflict of interest?

(A) No, because there was no breach of trust.

(B) No, because there was a full disclosure of the conflict.

(C) Yes, because there was no informed, written consent.

(D) Yes, because there was a conflict of interest to which the defendant
could not consent.

Module 4 Question 2

A husband and wife decide to divorce. The husband suggests that they use
the same lawyer to save money. After some disagreement over the property
settlement, they meet with an attorney and advise him that they cannot
agree on the property settlement and several tax issues, but that they are
willing to negotiate. The lawyer advises them of the potential conflict of
interest in representing both of them and that the case could result in a
contested proceeding based on their current disagreements. The husband and
wife sign a conflict waiver form. Because the divorce settlement will also
result in a substantial tax savings, the lawyer offers to take the case on
a contingent-fee basis, rather than an hourly fee.

Is the lawyer subject to disciplinary action?

(A) No, because the husband and wife were fully apprised of the conflict
of interest and waived it in writing.

(B) No, because contingent fee arrangements are acceptable in civil cases
as long as they are in writing and signed by the parties.

(C) Yes, because the lawyer could not obtain a valid waiver under these
circumstances.

(D) Yes, because the lawyer should have advised both parties to seek ad-
vice from independent counsel before signing the waiver.
Module 4 Question 3

A well-respected criminal defense lawyer is in such great demand that she


must turn down many prospective clients who are willing to pay her high
fees. She has been asked to represent four young men who were arrested for
possession of a substantial amount of drugs found in their car after po-
lice stopped them for having a broken tail light. There are several poten-
tial grounds to challenge the arrest, including illegal search and seizure
and denial of possession of the drugs. None of the young men could afford
the criminal defense lawyer by themselves, but they can if they pool their
funds. After interviewing all four, she is convinced that three of the de-
fendants have criminal exposure, but the fourth is completely innocent.
She decides to accept the case for the three defendants pro bono because
of the massive publicity the case is receiving in the newspapers and on
television. However, she advises the fourth defendant to meet with the
prosecutor to cooperate.

Is it proper for the criminal defense lawyer to represent the defendants?

(A) Yes, because the criminal defense lawyer may represent the three re-
maining defendants if she reasonably believes she can do so competently.

(B) Yes, because she properly advised the "innocent" defendant to cooper-
ate and did not violate any ethical rules.

(C) No, because the potential conflicts became actual conflicts that would
impair her ability to zealously represent the remaining defendants at
trial.

(D) No, because a criminal defense attorney can never represent more than
one defendant in an action.

Module 5 Question 1

An attorney runs a busy law firm that charges lower fees than do all other
lawyers in his vicinity. He is able to charge low fees because he requires
all of his clients to sign a contract waiving all possible malpractice
claims. Before having the clients sign the contract, the attorney advises
the clients that they should seek independent representation to help them
better understand the agreement. He also advertises his practice exten-
sively on the Internet.

Is the attorney is subject to discipline?

(A) Yes, because he requires a prospective waiver of all malpractice


claims.

(B) Yes, because malpractice claims are never prospectively waivable.

(C) No, because an attorney may choose to charge less than the prevailing
local amount.

(D) No, because the attorney advised the clients that they should seek in-
dependent representation before signing the malpractice waiver.
Module 5 Question 2

An attorney has just been admitted to practice law. The court would like
to appoint him to represent a defendant in a bigamy prosecution. The at-
torney believes that he is incompetent to handle the case because he
barely passed criminal procedure in law school and still does not under-
stand the topic. In addition, the attorney believes that bigamy should be
a capital offense. The attorney refuses the appointment after asking an-
other lawyer whether she would be interested in the appointment instead.

Is the attorney subject to discipline?

(A) Yes, because an attorney should not refuse a court appointment except
under very rare circumstances.

(B) Yes, because he did not try to associate himself with competent co-
counsel rather than refusing the appointment.

(C) No, because an attorney may refuse an appointment if he is incompetent


to handle the case or if his distaste for the case would impair his per-
formance.

(D) No, because the attorney attempted to engage substitute counsel before
he refused the court appointment.
Module 6 Question 1

A taxi driver was arrested for carrying a loaded gun in his cab, which is
a felony in this jurisdiction. The taxi driver hired a lawyer. Although
she had no idea on what basis to do so, the lawyer decided to file a mo-
tion to dismiss. She reserved a hearing date and time and notified the
prosecution of her intention to make the motion. She researched the rele-
vant statutes and state and federal case law, but found no legal authority
to support her motion. Then she reviewed the police report to see if there
was a factual basis to challenge the prosecution, but found none. The law-
yer decided to argue that the law inadvertently encouraged discrimination
against poor people because unprotected cab drivers are less likely to
drop off or pick up passengers in poor neighborhoods. She doubted her mo-
tion would succeed.

Should the lawyer be subject to discipline?

(A) Yes, because a lawyer shall not bring or defend a proceeding, or as-
sert or controvert an issue, unless she has a nonfrivolous basis in law
and fact for doing so.

(B) Yes, because a lawyer may not bring a motion for which her only argu-
ment is to reverse the law when the lawyer does not believe that the argu-
ment will succeed.

(C) No, because a lawyer for the defendant in a criminal proceeding may
defend the proceeding as to require that every element of the case be es-
tablished.

(D) No, because a lawyer is permitted to bring a motion for which her only
argument is that the law should be changed, even if the lawyer believes it
is unlikely to succeed.
Module 6 Question 2

A driver had standard liability coverage with an insurer. The policy pro-
vided that the insurer would provide a lawyer to defend the driver in
suits arising out of accidents, and it would pay for any settlement or
judgment against the driver. In addition, the driver would consent to the
insurer deciding whether to settle or try any suits.

The driver struck a pedestrian at an intersection, and a lawyer provided


by the insurer is representing the driver. The police report states that
the driver was not wearing glasses or contacts at the time. The driver had
claimed on his insurance application that he had 20/20 vision without
glasses or contacts. The insurer decides to investigate the issue because,
if the driver lied on the application, then the policy would be void and
the insurer would not be responsible.

The insurer informs the lawyer for the driver of the investigation and the
consequences to the driver for lying on the application. The insurer asks
the lawyer whether she noticed if the driver wore glasses or knew whether
the driver wore contacts. The insurer reminds the lawyer that she cannot
assist in a client's fraud. The lawyer feels sorry for the driver, an el-
derly man, and refuses to answer the insurer's questions.

Is the lawyer subject to discipline?

(A) Yes, because the lawyer owes a fiduciary duty to the insurer as a
third-party payor to provide the insurer with all information relating to
the representation.

(B) Yes, because the driver consented to have the insurer decide whether
to settle or try lawsuits, making the insurer the client to whom the law-
yer owes a duty of candor.

(C) No, because the lawyer cannot discuss any aspect of the driver's case
with the insurer, as the driver is the client and the insurer is only a
third-party payer to whom the lawyer owes no fiduciary duty.

(D) No, because the lawyer is not required to reveal information relating
to the representation of the driver, because the insurer is only a third-
party payor.
Module 6 Question 3

An attorney's 73-year-old client dies suddenly leaving his two daughters


as the only living relatives. The client's will leaves all of his consid-
erable estate to the young woman who walks his dog. The daughters have de-
cided to contest the will. At the trial, the attorney is contacted by the
press on his way into the courthouse. He states, "The daughters of my cli-
ent allege that his will was written under duress, but our answer details
several conversations that my client had with neighbors and friends in
which he firmly stated that he wanted to leave his estate to the dog
walker because she was such a sweet girl."

Has the attorney violated the Model Rules of Professional Conduct?

(A) No, because a lawyer may publicly state matters that are part of the
public record.

(B) No, because discovery had concluded.

(C) Yes, because trial publicity is not permitted regarding expected tes-
timony.

(D) Yes, because trial publicity is not permitted regarding the character
of a witness.
Module 7 Question 1

A seller and buyer are represented by separate counsel for the sale of
real property. During the course of negotiations, the buyer calls the
seller's lawyer to discuss some of the particulars of the real property.
They discuss the tax implications of the contract, and the lawyer provides
details of a group legal plan she could offer employees of the business
that the buyer intends to move to the subject property.

Is the lawyer subject to discipline for her communications with the buyer?

(A) Yes, because she has engaged in direct solicitation of professional


employment for her own pecuniary gain.

(B) Yes, because she has communicated with a represented party about the
matter that is the subject of the representation.

(C) No, because the buyer initiated contact with the lawyer.

(D) No, because the issues discussed were not related to a matter in an-
ticipation of litigation.
Module 8 Question 1

Two individuals involved in a dispute agree to have an attorney serve as a


mediator to help resolve the issue. The attorney meets with the two unrep-
resented parties individually and together during the mediation, but he is
unable to mediate a settlement. During one of the individual meetings, one
of the parties commented that she was "glad to have the attorney represent
her during the mediation." At no time did the attorney tell the parties he
was acting solely as a neutral mediator or that he did not represent ei-
ther party in an individual capacity. During a deposition in a subsequent
trial on the matter, the attorney was subpoenaed and testified to state-
ments made to him privately by the parties during the mediation.

Is the attorney subject to discipline?

(A) No, because the attorney was acting in a non-attorney capacity and
thus not subject to disciplinary rules.

(B) No, because the attorney was not representing either party, and thus
no conflict of interest problems existed.

(C) Yes, because the attorney's actions violated the attorney-client priv-
ilege.

(D) Yes, because the attorney had a duty to inform the parties that he was
not representing them in an individual capacity.
Module 8 Question 2

A retired butcher purchased a small condominium from a general contractor.


Shortly after he moved in, he discovered numerous defects, such as mold
and water leaks, on the premises. After several months of unsuccessful at-
tempts to get the contractor to fix the problems, the butcher filed a pro
se suit. At the first status conference, the judge ordered the butcher and
the contractor's attorney to participate in the court's mediation service
before proceeding further. At the mediation, the state mediator asked the
butcher to explain his case to her first, outside the presence of the con-
tractor's counsel, then asked the contractor's counsel to do the same. At
a joint session following the private meetings, the contractor's attorney
told the butcher he would never be able to win and he should take a small
settlement. Then, in another private meeting, the mediator told the
butcher, "The contractor's attorney is right. You're a retired butcher
who's never been in a lawsuit. You could end up with nothing." The butcher
asked, "Do you really think that's best?" The mediator replied, "I do."
Based on her advice, the butcher settled the case for a nominal sum.

Is the mediator likely to be disciplined?

(A) No, because she met with the butcher privately, did not reveal to the
contractor's counsel what she told him, and held a joint session of all
parties to discuss settlement.

(B) No, because she simply reminded the butcher that he was a retired
butcher, explained to him that he could end up with nothing, and advised
him that she thought accepting the offer was best.

(C) Yes, because she failed to explain to the butcher that she didn't rep-
resent him and didn't tell him the difference between a mediator and a
lawyer representing a client.

(D) Yes, because she stepped out of the role of neutral third party and
gave the butcher explicit legal advice, and as a result, he settled the
case for what little was offered.
Module 9 Question 1

A lawyer agrees to represent a personal injury plaintiff on a contingent


fee basis. The fee agreement specifies that the attorney will be entitled
to 30% of any recovery. The lawyer obtains a $200,000 verdict. The attor-
ney advanced $50,000 in costs over the course of the litigation. The at-
torney deposits all $200,000 into an appropriate trust account and con-
tacts the client immediately regarding disbursement of the money. The
lawyer advises the client that he will take $60,000 in fees (30% of
$200,000) and $50,000 in reimbursement for costs advanced. As a result,
the client will receive a check for $90,000. The client argues that the
attorney should take his 30% fee from the amount of the verdict after ex-
penses have been deducted. The client does not dispute that the attorney
is entitled to reimbursement of the $50,000, but he argues that the law-
yer's fee should be $45,000 (30% of the remaining $150,000) and that the
client is entitled to $105,000. The fee agreement is silent as to whether
the attorney's fee should be computed based on the amount obtained before
or after costs are deducted.

How much money must the attorney keep in the trust account while the at-
torney and the client resolve their dispute?

(A) $200,000.

(B) $150,000.

(C) $105,000.

(D) $15,000.

Module 9 Question 2

A lawyer represented a client in complex and lengthy litigation. The law-


yer received a $4 million check in payment of the judgment the lawyer ob-
tained for the client, which the lawyer deposited in his client trust ac-
count. The lawyer and the client then had a dispute over how much money
the lawyer is owed in fees and costs.

Which of the following actions is neither recommended nor required by the


Model Rules?

(A) Paying the client the amount that is not in dispute.

(B) Suggesting a means for prompt resolution of the dispute.

(C) Withdrawing from the client trust account the amount of fees and costs
that are not in dispute.

(D) Notifying the appropriate State Bar of the dispute.


Module 10 Question 1

A lawyer who specializes in divorce and custody law realizes that there
are very few family lawyers on the east side of town. She also notices a
newspaper article in the local paper describing the high rate of divorce
in communities on the east side of town. In an effort to reach as many
potential clients as possible, she researches court records to locate pro
se divorce filings and prepares an advertising circular containing
information about her office and her practice to mail to all
unrepresented divorce litigants. She mails the circular to them in an
envelope with her name and return address.

Did the attorney act properly?


(A) Yes, because a lawyer may advertise to a pro se litigant as a
service to the litigant.

(B) Yes, because mailings cannot be targeted to those known to be in need


of a particular legal service.

(C) No, because the mailings targeted those known to be in need of a


particular legal service.

(D) No, because she must wait until 30 days after a pro se litigant files
before contacting him or her.

Module 10 Question 2

While driving home, a lawyer sees a three-car collision. There are no po-
lice or ambulances at the scene and it appears the accident has just oc-
curred. The lawyer, who is also a licensed medical doctor, pulls her car
to the side of the road, calls 911 on her cell phone and then grabs her
first aid kit and tries to assist the people involved in the accident.
While bandaging a bleeding woman, the clearly traumatized woman exclaims
repeatedly, "That jerk! He was talking on his cell phone and ran a red
light. What am I going to do? I can't afford this!" The lawyer tells the
woman, "I'm a doctor but I'm also a lawyer. I just wanted to let you know
that you should obtain legal representation as soon as possible. You can
call me if you have any questions, free of charge." The lawyer then gives
the woman her card.

Was the lawyer's conduct proper?

(A) Yes, because the woman was clearly in need of legal services and asked
for advice.

(B) Yes, because attorneys are permitted to advise people of the need to
obtain legal counsel.

(C) No, because in-person solicitation of a specific prospective client is


completely prohibited under the Model Rules.

(D) No, because the woman was clearly traumatized and under duress.
Module 11 Question 1

A district attorney asked a grand jury to indict a contractor for bribery


and racketeering. The district attorney does not have probable cause to
believe that the contractor actually committed the crimes, and does not
intend to prosecute the contractor, but he wants to use the indictment as
leverage to try to get him to testify against his friends, who are the
leaders of a major drug-trafficking organization.

Should the district attorney be disciplined for this conduct?

(A) No, because inducing the contractor to testify would be "in the inter-
est of justice."

(B) No, because he does not intend to prosecute the contractor.

(C) Yes, because attorneys may not present false statements or evidence
before a tribunal.

(D) Yes, because he doesn't have sufficient evidence to support an indict-


ment.
Module 11 Question 2

A community suffered several home invasions in which the perpetrator tied


up the occupants and ransacked the homes for cash and jewelry. The entire
city was on edge until the perpetrator was caught outside a home that had
just been ransacked. Several eyewitnesses also identified him in a police
line-up as the person they had seen leaving other homes that had been in-
vaded. The prosecutor told the media at a press conference: "Due to dili-
gent efforts of our city's finest, we have the perpetrator of these des-
picable acts. We congratulate the police on this arrest. These crimes ter-
rorized our city, and we will prosecute this guilty perpetrator to the
fullest extent of the law."

The suspect was held without bail. During the course of the prosecutor's
pretrial preparations, he learned that the suspect is the boyfriend of a
teenage girl who lives in the last home invaded. Furthermore, the suspect
had a solid alibi for the times at which two other home invasions took
place. The prosecutor duly passed all this information along to the de-
fense attorney. After more investigation, the prosecutor dropped the
charges.

Is the prosecutor subject to discipline?

(A) No, because the prosecutor promptly provided exculpatory information


to defense counsel.

(B) No, because the prosecutor dismissed the charges when he determined
that evidence was lacking.

(C) Yes, because the prosecutor's comments to the media were prohibited.

(D) Yes, because the prosecutor initiated a prosecution without probable


cause.
Module 12 Question 1

A clerk has been clerking for a judge for the past three years. In the
past, the judge and the clerk have not always seen eye to eye on both per-
sonal and legal issues. When a position opens with another judge in the
court, the clerk decides she would like to take the position. The clerk
asks the judge for whom she currently clerks for a letter of recommenda-
tion and the judge agrees, knowing her recommendation will hold clout with
the other judge. The judge is also excited at the prospect of getting a
new clerk for herself. She writes a letter of recommendation for the
clerk on her official letterhead.

Will the judge be subject to discipline for writing the letter of recom-
mendation for her clerk?

(A) Yes, because the judge may not use her name, influence, or the pres-
tige of her office for this purpose.

(B) Yes, because the judge knew that the recommendation on her letterhead
would hold clout with the other judge.

(C) No, because the only requirement for a judge to use her letterhead for
a recommendation is that the recommendation is based on the judge's per-
sonal knowledge.

(D) No, because a judge may provide a reference such as the one in this
case on her official letterhead.

Module 12 Question 2

A judicial clerk is acting as the judge's primary research clerk for a


complex litigation. A partner for one of the law firms involved is very
impressed by the clerk's work and wants to hire her as an associate to
work on cases other than the litigation before the judge. The clerk noti-
fies the judge and then proceeds to negotiate with the partner over her
terms of employment as an associate with the partner's firm.

Is the clerk's conduct proper?

(A) Yes, because she notified the judge before negotiating employment with
the firm.

(B) Yes, because a law clerk is not bound by the restrictions precluding
judges from negotiating for employment with lawyers involved in matters in
which the judge is participating personally and substantially.

(C) No, because a law clerk may not negotiate for employment with a party
or lawyer who is involved in a matter in which the law clerk is partici-
pating personally and substantially.

(D) No, because the partner and the partner's law firm did not withdraw
from representation in the complex litigation before the judge.
After completing the quiz for each module, review your answers with the Question Based
Review video on your online syllabus. The full answer explanations are below as well.

Module 1 Question 1
A young associate works for a small plaintiffs' personal injury firm. One
afternoon, she is driving from a deposition back to the office when she
encounters a sudden stop in traffic. She calls her supervising partner to
advise that she will be late getting back to the office. While the two
are talking on the phone, the young associate notices that the cause for
the traffic jam is a four-car accident up ahead. She sees several injured
people on the side of the road and mentions this to the partner. The
partner tells the associate to pull over and give business cards to the
injured people. Knowing this is an ethical violation, the associate says
she does not feel comfortable doing so. The partner responds, "I'm making
a judgment call here. Go ahead and hand out your business card. If you
get in trouble for this, I'll handle it." The young associate does indeed
hand out business cards to the parties involved in accident. Assume that
the court finds that this is an ethical violation.

Will the young associate be subject to discipline for her actions?

(A) No, because the young associate was following the orders of the
supervisory attorney.

(B) No, because the young associate acted in accordance with a supervisory
lawyer's reasonable resolution of an arguable question of professional
duty.

(C) Yes, because following a supervisor's orders is never a defense to an


ethical violation of a subordinate lawyer.

(D) Yes, because the young associate knew it was a violation to hand out
business cards at the scene of the accident.

The correct answer is: (D) Yes, because the young associate knew it was a
violation to hand out business cards at the scene of the accident.

Where a lawyer has committed an ethical violation, it is generally no de-


fense that the attorney acted in accordance with a supervising attorney's
orders. The subordinate lawyer has his own ethical duties, and he is obli-
gated to act in accordance with these duties. MR 5.1. However, where the
subordinate seeks the guidance of a supervisory lawyer on an arguable
question of a professional duty, the subordinate may act in accordance
with the supervisor's reasonable resolution of the matter. In this case,
there was no arguable question of professional duty. The young associate
that she should not hand out business cards to the injured parties on the
side of the road. The fact that the supervisory attorney told her to go
ahead and hand out the business cards will not serve as a defense to the
young attorney, and she will be subject to discipline. Note that the su-
pervising lawyer will also be subject to discipline.

(A) Incorrect. No, because the young associate was following the orders of
the supervisory attorney.
When a subordinate lawyer has committed an ethical violation, it is gener-
ally no defense that the attorney acted in accordance with a supervising
lawyer's orders. The subordinate has his own ethical duties, and he is ob-
ligated to act in accordance with these duties. An exception exists where
a subordinate attorney acts in accordance with a supervisory lawyer's rea-
sonable resolution of an arguable question of professional duty, in which
case the subordinate attorney will not be subject to discipline for an
ethical violation. However, this case does not present an arguable ques-
tion of professional duty. The young associate knew that handing out busi-
ness cards was an ethical violation. Even though she was following her
supervising attorney's orders, this is no defense to a violation of the
young associate's own ethical duties. Therefore, this answer choice is
incorrect.

(B) Incorrect. No, because the young associate acted in accordance with a
supervisory lawyer's reasonable resolution of an arguable question of pro-
fessional duty.

It is true that where a subordinate attorney acts in accordance with a su-


pervisory lawyer's reasonable resolution of an arguable question of pro-
fessional duty, the subordinate attorney will not be subject to discipline
for an ethical violation. However, this case does not present an arguable
question of professional duty. The young associate handing out business
cards was an ethical violation. Even though she was following her super-
vising attorney's orders, this is no defense to a violation of the young
associate's own ethical duties. Therefore, this answer choice is incor-
rect.

(C) Incorrect. Yes, because following a supervisor's orders is never a de-


fense to an ethical violation of a subordinate lawyer.

This answer choice is too broad. Generally, the fact that a lawyer was
following a supervisor's orders is not a defense to an ethical violation.
The exception is when the subordinate lawyer seeks the guidance of a su-
pervisory lawyer on an arguable question of a professional duty, in which
case the subordinate may act in accordance with the supervisor's reasona-
ble resolution of the matter without fear of disciplinary action. Here,
there is no arguable question of professional duty. The young associate
knew she should not hand out her business cards, but she did it anyway.
Her own ethical duties remain, and she cannot escape disciplinary action
by arguing that she was acting on her supervisor's orders. Note that the
supervising lawyer will also be subject to discipline.

Module 1 Question 2

Two attorneys were the sole partners in a law firm. Their office manager
was not a lawyer. The office manager had three friends who were seriously
injured in an automobile accident. The office manager encouraged his three
friends to hire the law firm to represent the friends in a lawsuit to re-
cover damages for the friends' injuries. The two attorneys were able to
negotiate a multimillion dollar settlement for the office manager's
friends, which resulted in substantial attorney fees for the firm. The two
attorneys were grateful to the office manager for referring his friends to
the firm. Under the law firm's profit-sharing plan, the office manager re-
ceived one percent of the attorney fee that the two attorneys received for
representing the office manager's friends.

Is the payment to the office manager proper?

(A) Yes, because the office manager referred the clients to the two attor-
neys.

(B) Yes, because the payment is pursuant to the firm's profit-sharing


plan.

(C) No, because the office manager is not a lawyer.

(D) No, because the office manager solicited his friends to hire the two
attorneys.

The correct answer is: (B) Yes, because the payment is pursuant to the
firm's profit-sharing plan.

Although Model Rule 5.4(a) prohibits the sharing of fees between lawyers
and nonlawyers, Model Rule 5.4(a)(3) provides that a lawyer or law firm
may include nonlawyer employees in a compensation or retirement plan, even
though the plan is based in whole or in part on a profit-sharing arrange-
ment. The office manager is an employee of the attorneys, and the office
manager received the payment under the firm's profit-sharing plan. As long
as the office manager receives the payment as part of his compensation un-
der the profit-sharing plan, and not as some kind of referral bonus, the
payment is permissible under Model Rule 5.4(a)(3). Thus, this answer is
correct.

(A) Incorrect. Yes, because the office manager referred the clients to the
two attorneys.

This answer choice is incorrect because, in general, the payment of a re-


ferral fee to the office manager, who is a nonlawyer employee, would be
prohibited by Model Rule 5.4(a), as noted above. Had the answer choice
reasoning explained that the office manager received the payment under the
firm's profit-sharing plan, which is permissible under Model Rule
5.4(a)(3), the answer choice would have been correct.

(C) Incorrect. No, because the office manager is not a lawyer.

This answer choice is incorrect because pursuant to Model Rule 5.4(a)(3),


the payment to the office manager, who is a nonlawyer, would be permissi-
ble if made under the firm's profit-sharing plan. Pursuant to that provi-
sion, as long as the office manager receives the payment as part of his
compensation under the profit-sharing plan, and not as some kind of refer-
ral bonus, the payment is permissible. Thus, the fact that the office man-
ager is not a lawyer would be irrelevant and would render this answer
choice incorrect.
(D) Incorrect. No, because the office manager solicited his friends to
hire the two attorneys.

This answer choice implicates the rules against solicitation. Model Rule
7.3(a) provides that a lawyer shall not, by in-person, live telephone, or
real-time electronic contact, solicit professional employment from a pro-
spective client when a significant motive for the lawyer's doing so is the
lawyer's pecuniary gain, unless the person contacted: (1) is a lawyer; or
(2) has a family, close personal, or prior professional relationship with
the lawyer. Here, the two attorneys did not directly solicit the office
manager's friends and did not direct the office manager to solicit them as
clients. Moreover, the office manager did not solicit strangers on behalf
of the two attorneys. He already had a close personal relationship with
the prospective clients; they were his friends. If the office manager were
a lawyer, the office manager would not have violated his ethical obliga-
tions by soliciting his friends on behalf of the law firm. Thus, this an-
swer choice is incorrect.

Module 2 Question 1

An attorney practices business law and regularly incorporates small busi-


nesses. Several years ago, the attorney incorporated a gift shop for a
client. She has not heard from that client since the incorporation was
completed. The attorney learns of new business regulations that she be-
lieves would affect the gift shop owner's business.

Would the attorney be subject to discipline if she calls the gift shop
owner to advise her of the new regulations?

(A) No. In fact, the attorney would be subject to discipline if she did
not contact the gift shop owner about the new regulations.

(B) No, because the gift shop owner has a prior professional relationship
with the attorney.

(C) Yes, because the gift shop owner is no longer the attorney's client.

(D) Yes, although the attorney would be permitted to send the gift shop
owner a letter regarding the new regulations.

The correct answer is: (B) No, because the gift shop owner has a prior
professional relationship with the attorney.

The Rules prohibit live, real-time contact with prospective fee-generating


clients unless those prospective clients are lawyers, relatives, close
friends or former clients. The types of communication prohibited are in-
person or real-time electronic contact (e.g., phoning or sending text mes-
sages). Real-time contacts, such as phone calls, are believed to be more
coercive because of their immediacy. The attorney in this case would not
be subject to discipline for phoning the gift shop owner because the gift
shop owner is a former client.
(A) Incorrect. No. In fact, the attorney would be subject to discipline if
she did not contact the gift shop owner about the new regulations.

The Model Rules do not require that the attorney contact her former client
regarding the new regulations. The gift shop owner has not been a client
for years, so there can be no expectation on the part of the gift shop
owner that the attorney is continuing to look out for the gift shop own-
er's legal interests. In spite of the years since representation, the at-
torney is permitted to contact her former client by phone to inform the
former client about the new regulations that may affect her business. For-
mer clients, along with other attorneys, close friends, and family mem-
bers, are exempt from the prohibition of real-time contact with prospec-
tive fee-generating clients.

(C) Incorrect. Yes, because the gift shop owner is no longer the attor-
ney's client.

This answer choice is incorrect, because the Rules specifically permit


lawyers to contact former clients by phone (or other real-time contact)
for the purpose of generating additional fees. The only way the attorney
in this case would face discipline for phoning her former client is if the
former client had expressly forbidden the attorney from contacting her.

(D) Incorrect. Yes, although the attorney would be permitted to send the
gift shop owner a letter regarding the new regulations.

The attorney may contact the gift shop owner because she is a former cli-
ent. The Rules permit the attorney to contact the former client by phone
or other methods both immediate and not, including letter writing. This
answer choice is incorrect, because the attorney would not be subject to
discipline for phoning her former client.

Module 2 Question 2

A client asks his attorney to negotiate a sales contract. The client is


pressed for time and offers the attorney twice her normal rate to expedite
the request. The attorney is very busy but thinks the client will pay even
more to compensate her. Over the phone, the attorney explains her workload
predicament to her client and makes a counter-offer to negotiate the sales
contract for a percentage of the sales price. The client agrees.

For which of the following could the attorney be subject to discipline?

(A) Failing to put the fee agreement in writing.

(B) Profiting excessively from a time-disadvantaged client.

(C) Accepting the employment, because doing so would preclude other em-
ployment by the attorney.

(D) The fee, which is unreasonably high for this service.


The correct answer is: (A) Failing to put the fee agreement in writing.

The Model Rules generally do not require a written fee arrangement. How-
ever, a contingent fee agreement must be written. Therefore, this is the
correct answer.

(B) Incorrect. Profiting excessively from a time-disadvantaged client.

A lawyer shall not charge an unreasonable fee under the Model Rules. The
factors to be considered in determining the reasonableness of a fee in-
clude time limitations imposed by the client. In this case, the client of-
fered more for expedited service, and the client accepted the attorney's
counter-offer. The problem in this situation is that the attorney did not
capture the agreement in writing as is required for contingent fees. Thus,
this answer is incorrect.

(C) Incorrect. Accepting the employment, because doing so would preclude


other employment by the attorney.

This is a misstatement of one of the factors determining the reasonable-


ness of a fee. The factor suggests a lawyer may adjust her fee for a job
if taking on the job would monopolize her time. Thus, this answer is not
correct.

(D) Incorrect. The fee, which is unreasonably high for this service.

A lawyer may not charge an unreasonable fee. However, there are many fac-
tors that determine whether a fee is reasonable. In the case presented,
the fee would probably not be considered unreasonable because the client
offered to pay for expedited services and the attorney, being very busy,
may have incurred additional expenses to properly serve her other clients
in order to take on the contract negotiation. This is therefore not the
best answer.

Module 3 Question 1

An electrician and a lawyer went to high school together. One afternoon,


the electrician comes to the lawyer's office. He sits down, pulls a gun
out of his pocket, and tells the lawyer, "I shot my wife this morning."
The lawyer quickly tells the electrician to put the gun back in his pocket
and to say nothing more. He then tells the electrician that his practice
is limited to tax and real estate matters, that he is not qualified to
represent the electrician or give competent legal advice, and that the
electrician should seek representation from a good criminal defense attor-
ney. The lawyer hands the electrician a list of competent criminal defense
attorneys and asks him to leave. The electrician is arrested and charged
with murder. The lawyer is arrested for evidence tampering after the elec-
trician tells the police that the lawyer told him to hide the gun.

Would the lawyer be subject to discipline if he reveals that he only told


the electrician to put the gun back in his pocket?
(A) Yes, because the lawyer's duty of confidentiality prohibits him from
disclosing his conversations with the electrician.

(B) Yes, if the disclosure would be detrimental to the electrician's in-


terests.

(C) No, if the disclosure is necessary to defend against the evidence-tam-


pering charge.

(D) No, because the lawyer declined to represent the electrician, so the
attorney-client privilege does not apply.

The correct answer is: (C) No, if the disclosure is necessary to defend
against the evidence-tampering charge.

Model Rule 1.6(a) regarding confidentiality of information bars a lawyer


from revealing information relating to the representation of a client.
However, there are exceptions to this rule. For example, under Model Rule
1.6(b)(5), a lawyer is permitted to reveal confidential client communica-
tions to establish a defense to a criminal charge against the lawyer based
on conduct in which the client was involved. However, even under this ex-
ception, the lawyer should only reveal such portions of confidential cli-
ent communications that are necessary to establish the defense to the ac-
cusation of wrongdoing. Thus, this choice is correct. The lawyer can re-
veal that he merely told the electrician to put the gun back in his pocket
in order to defend himself against the evidence-tampering charge. However,
he should not reveal anything else about his encounter with the electri-
cian that morning.

(A) Incorrect. Yes, because the lawyer's duty of confidentiality prohibits


him from disclosing his conversations with the electrician.

Lawyers cannot reveal confidential information relating to the representa-


tion of a client [MR 1.6]. However, one exception is that a lawyer may re-
veal confidential client communications to establish a defense for himself
against a criminal charge based on conduct in which the client was in-
volved. Therefore, this is not the best choice because the lawyer is per-
mitted to reveal the electrician's confidences when an exception to Model
Rule 1.6 applies, as it does here.

(B) Incorrect. Yes, if the disclosure would be detrimental to the electri-


cian's interests.

This is not the best answer choice because, even though the disclosure
might be detrimental to the electrician's interests, it is still permissi-
ble under the Model Rules so that the lawyer may defend himself against a
criminal charge.

(D) Incorrect. No, because the lawyer declined to represent the electri-
cian, so the attorney-client privilege does not apply.

Even when no client-lawyer relationship ensues, a lawyer who has had dis-
cussions with a prospective client shall not use or reveal information
learned in the consultation [MR 1.18]. Thus, the attorney-client privilege
does apply to the lawyer's encounter with the electrician. However, an ex-
ception to the confidentiality rule applies in this case where the lawyer
must disclose part of the conversation to defend himself against a crimi-
nal charge. Thus, this answer is incorrect.

Module 3 Question 2

A business owner has been sued for violations of the Americans With Disa-
bilities Act. He calls his friend, who is a lawyer, to ask if the friend
could represent him in the case. In the course of the conversation, he
confided in his lawyer friend about an unrelated legal matter, in which he
was represented by another attorney. Because the lawyer friend had no ex-
pertise or experience with ADA cases, he declined the representation. Sub-
sequently, the lawyer friend is called as a witness in the unrelated legal
matter and agrees to disclose the substance of his conversation with the
business owner over the business owner's objection.

Is the lawyer friend's action proper?

(A) Yes, because the lawyer friend could not competently represent him in
the Americans With Disabilities Act case.

(B) Yes, but only the portion of the conversation regarding the legal mat-
ter unrelated to the initial consultation.

(C) No, because the entire conversation is confidential.

(D) No, unless the lawyer believes disclosure is in the business owner's
interest.

The correct answer is: (C) No, because the entire conversation is confi-
dential.

Model Rule 1.18 prohibits a lawyer from revealing information learned in a


consultation with a prospective client, even when no client-lawyer rela-
tionship ensues. Here, the business owner called the lawyer, seeking legal
representation. As a prospective client, he is owed a duty of confidenti-
ality by the lawyer, and the lawyer may not disclose the substance of the
conversation.

(A) Incorrect. Yes, because the lawyer friend could not competently repre-
sent him in the Americans With Disabilities Act case.

Model Rule 1.18 prohibits a lawyer from revealing information learned in a


consultation with a prospective client, even when no client-lawyer rela-
tionship ensues. Here, the business owner was a prospective client and is
owed a duty of confidentiality even though the lawyer could not compe-
tently represent him and declined the representation. Thus, this answer is
incorrect.

(B) Incorrect. Yes, but only the portion of the conversation regarding the
legal matter unrelated to the initial consultation.
Model Rule 1.18 governs an attorney's duty of confidentiality toward a
prospective client and prohibits an attorney from revealing information
learned in the consultation with the prospective client, even when no cli-
ent-lawyer relationship ensues. Here, the business owner called the law-
yer, seeking legal representation. As a prospective client, he is owed a
duty of confidentiality by the lawyer, and the lawyer may not be compelled
to disclose the substance of the conversation. Even though the lawyer
could not competently represent the business owner and so declined the
representation, that fact alone does not prevent the conversation from be-
ing considered confidential. Moreover, even though the business owner was
not seeking legal representation with respect to the unrelated matter, MR
1.18 protects confidential communications that are unrelated to the sub-
ject of the consultation. Thus, this answer is incorrect.

(D) Incorrect. No, unless the lawyer believes disclosure is in the busi-
ness owner's interest.

This choice uses an inapplicable standard and is thus incorrect. Model


Rule 1.18 governs an attorney's duty of confidentiality toward a prospec-
tive client and prohibits an attorney from revealing information learned
in the consultation with the prospective client, even when no client-law-
yer relationship ensues. There are certain exceptions to the attorney-cli-
ent privilege under MR 1.6, but disclosure because the lawyer believes it
is in the client's interest is not one of them.

Module 4 Question 1

A defendant is charged with grand larceny. She tells an attorney that she
is innocent and asks the attorney to defend her. As it turns out, the at-
torney's sister is handling the defendant's prosecution. The attorney dis-
closes to the defendant that her sister is the prosecutor and then asks to
hear the defendant's side of the story. The attorney agrees to represent
the defendant. During the course of the prosecution, the attorney does not
talk to her sister about the case or anything else.

Is the attorney subject to discipline for a conflict of interest?

(A) No, because there was no breach of trust.

(B) No, because there was a full disclosure of the conflict.

(C) Yes, because there was no informed, written consent.

(D) Yes, because there was a conflict of interest to which the defendant
could not consent.

The correct answer is: (C) Yes, because there was no informed, written
consent.

The Rules require that where opposing counsel are related, each client
must give informed consent confirmed in writing to the representation by
their attorney. In this case, where the opposing counsel in a prosecution
are sisters, there is no indication that the defendant's attorney fully
informed the client about the impact the conflict might have on her case
or any available alternatives. Furthermore, there is no indication that
the client's consent was confirmed in writing. Therefore, the attorney
would be subject to discipline under the Rules.

(A) Incorrect. No, because there was no breach of trust.

Breach of trust or not, the Rules require that the defendant's attorney
obtain informed consent, confirmed in writing, from the client before pro-
ceeding with her defense in a prosecution where the attorney's sister is
the prosecutor. Merely telling the client about the relationship is not
enough. Thus, this answer is incorrect.

(B) Incorrect. No, because there was a full disclosure of the conflict.

Although the defendant's attorney informed the defendant that her sister
was the prosecutor, there was not informed consent under the Rules. The
Rules require the attorney to provide adequate information and explanation
about the material risks involved as a result of the conflict, and to ex-
plain reasonable alternatives. Furthermore, the client's consent must be
confirmed in writing. Thus, this answer is incorrect.

(D) Incorrect. Yes, because there was a conflict of interest to which the
defendant could not consent.

There are situations under the Rules where a client cannot consent to rep-
resentation in the face of a conflict. For example, an attorney can't rep-
resent both the plaintiff and the defendant in the same lawsuit. Here, the
conflict arises over the close relation between opposing counsel. The cli-
ent may give informed consent, confirmed in writing, to accept representa-
tion by the attorney in this instance. Thus, this answer is incorrect.

Module 4 Question 2

A husband and wife decide to divorce. The husband suggests that they use
the same lawyer to save money. After some disagreement over the property
settlement, they meet with an attorney and advise him that they cannot
agree on the property settlement and several tax issues, but that they are
willing to negotiate. The lawyer advises them of the potential conflict of
interest in representing both of them and that the case could result in a
contested proceeding based on their current disagreements. The husband and
wife sign a conflict waiver form. Because the divorce settlement will also
result in a substantial tax savings, the lawyer offers to take the case on
a contingent-fee basis, rather than an hourly fee.

Is the lawyer subject to disciplinary action?

(A) No, because the husband and wife were fully apprised of the conflict
of interest and waived it in writing.

(B) No, because contingent fee arrangements are acceptable in civil cases
as long as they are in writing and signed by the parties.
(C) Yes, because the lawyer could not obtain a valid waiver under these
circumstances.

(D) Yes, because the lawyer should have advised both parties to seek ad-
vice from independent counsel before signing the waiver.

The correct answer is: (C) Yes, because the lawyer could not obtain a
valid waiver under these circumstances.

Some conflicts cannot be waived even with informed client consent. A law-
yer may not represent a client if the representation involves the asser-
tion of a claim by one client against another client represented by the
lawyer in the same litigation or other proceeding before a tribunal, even
with the clients' consent [MR 1.7]. Thus, even with both parties' consent,
the lawyer could not properly represent both of them in a contested di-
vorce case, which was probable if their differences could not be settled.
Even if the parties sought the advice of independent counsel before sign-
ing the waiver, the consent would be invalid.

(A) Incorrect. No, because the husband and wife were fully apprised of the
conflict of interest and waived it in writing.

Informed consent would not be sufficient here, even if the parties sought
independent advice before signing the waiver. A lawyer may not represent a
client if the representation involves the assertion of a claim by one cli-
ent against another client represented by the lawyer in the same litiga-
tion or other proceeding before a tribunal, even with the clients' consent
[MR 1.7]. This case was likely to become a contested divorce and, there-
fore, the conflict could not be waived. Thus, this answer is incorrect.

(B) Incorrect. No, because contingent fee arrangements are acceptable in


civil cases as long as they are in writing and signed by the parties.

A lawyer is prohibited from forming a contingent fee arrangement in a do-


mestic relations matter [MR 1.5]. Therefore, this is not the best answer.

(D) Incorrect. Yes, because the lawyer should have advised both parties to
seek advice from independent counsel before signing the waiver.

Even if the parties had sought independent advice, a waiver would not be
valid here. A lawyer may not represent a client if the representation in-
volves the assertion of a claim by one client against another client rep-
resented by the lawyer in the same litigation or other proceeding, even
with the clients' consent [MR 1.7]. This case was likely to become a con-
tested divorce and, therefore, the conflict could not be waived. Thus,
this answer is incorrect.

Module 4 Question 3

A well-respected criminal defense lawyer is in such great demand that she


must turn down many prospective clients who are willing to pay her high
fees. She has been asked to represent four young men who were arrested for
possession of a substantial amount of drugs found in their car after po-
lice stopped them for having a broken tail light. There are several poten-
tial grounds to challenge the arrest, including illegal search and seizure
and denial of possession of the drugs. None of the young men could afford
the criminal defense lawyer by themselves, but they can if they pool their
funds. After interviewing all four, she is convinced that three of the de-
fendants have criminal exposure, but the fourth is completely innocent.
She decides to accept the case for the three defendants pro bono because
of the massive publicity the case is receiving in the newspapers and on
television. However, she advises the fourth defendant to meet with the
prosecutor to cooperate.

Is it proper for the criminal defense lawyer to represent the defendants?

(A) Yes, because the criminal defense lawyer may represent the three re-
maining defendants if she reasonably believes she can do so competently.

(B) Yes, because she properly advised the "innocent" defendant to cooper-
ate and did not violate any ethical rules.

(C) No, because the potential conflicts became actual conflicts that would
impair her ability to zealously represent the remaining defendants at
trial.

(D) No, because a criminal defense attorney can never represent more than
one defendant in an action.

The correct answer is: (C) No, because the potential conflicts became ac-
tual conflicts that would impair her ability to zealously represent the
remaining defendants at trial.

There is a presumption against joint representation of criminal defend-


ants. As the comments to Model Rule 1.7 indicate, "The potential for con-
flict of interest in representing multiple defendants in a criminal case
is so grave that ordinarily a lawyer should decline to represent more
than one co-defendant." The potential for harm arising from conflicts
where liberty or life is at stake is greater in criminal cases than in
civil cases. Here, the criminal defense lawyer would be unable to cross-
examine the fourth defendant at trial, based on her interview with him
while they had an attorney-client relationship. The inability to use this
information impairs her ability to represent the remaining three defend-
ants. Therefore, she is now disqualified from representing the remaining
defendants.

(A) Incorrect. Yes, because the criminal defense lawyer may represent the
three remaining defendants if she reasonably believes she can do so compe-
tently.

In this situation, it would not be enough that the lawyer has a "reasona-
ble belief" that she can represent them. In general, it is more reasonable
to believe that joint representation is not possible in criminal cases
than otherwise because defendants are motivated to inform on co-defend-
ants and the like. The problem in this case is that, in telling the
fourth defendant to cooperate with prosecutors (which was perfectly ethi-
cal), the attorney has had private attorney-client conversations with
parties on "both sides." That is a conflict which is not subject to in-
formed consent [MR 1.7 [cmt. 23]].

(B) Incorrect. Yes, because she properly advised the "innocent" defendant
to cooperate and did not violate any ethical rules.

It is true that the lawyer's advice to the fourth defendant was proper.
The problem is that, in telling the fourth defendant to cooperate with
prosecutors, she is now in a position where she has had private attorney-
client conversations with parties on "both sides," and that cannot be
erased by anyone's consent [MR 1.7 [cmt. 23]]. Thus, this answer is incor-
rect.

(D) Incorrect. No, because a criminal defense attorney can never represent
more than one defendant in an action.

It is not correct that a criminal defense attorney can never represent


more than one defendant in an action. However, there is a presumption
against doing so because of the high stakes involved and the resultant mo-
tivations that defendants may have to testify against each other [MR 1.7
[cmt. 23]].

Module 5 Question 1

An attorney runs a busy law firm that charges lower fees than do all other
lawyers in his vicinity. He is able to charge low fees because he requires
all of his clients to sign a contract waiving all possible malpractice
claims. Before having the clients sign the contract, the attorney advises
the clients that they should seek independent representation to help them
better understand the agreement. He also advertises his practice exten-
sively on the Internet.

Is the attorney is subject to discipline?

(A) Yes, because he requires a prospective waiver of all malpractice


claims.

(B) Yes, because malpractice claims are never prospectively waivable.

(C) No, because an attorney may choose to charge less than the prevailing
local amount.

(D) No, because the attorney advised the clients that they should seek in-
dependent representation before signing the malpractice waiver.

The correct answer is: (A) Yes, because he requires a prospective waiver
of all malpractice claims.

Model Rule 1.8 states that a lawyer cannot make an agreement with a client
prospectively limiting the lawyer's malpractice liability unless the cli-
ent is independently represented in making the agreement. The attorney
routinely requires his clients to waive all prospective malpractice
claims, but does not apparently have the clients represented by independ-
ent counsel in making that waiver. Therefore, he is subject to discipline.

(B) Incorrect. Yes, because malpractice claims are never prospectively


waivable.

This is not a correct statement of the law. In fact, a client can prospec-
tively limit his right to sue an attorney for malpractice. However, he
must be represented by independent counsel in making the waiver. Thus,
this answer is incorrect.

(C) Incorrect. No, because an attorney may choose to charge less than the
prevailing local amount.

Model Rule 1.8 provides that a lawyer is not permitted to make an agree-
ment prospectively limiting the lawyer's liability to a client for mal-
practice unless the client is independently represented in making the
agreement. Thus, it is not enough that the attorney advised the clients
that they should seek independent representation; the clients must have
been independently represented in making the agreement. Thus, this answer
is incorrect.

(D) Incorrect. No, because the attorney advised the clients that they
should seek independent representation before signing the malpractice
waiver.

It is true that nothing in the question seems to show that the advertising
violates the Model Rules. However, the attorney could still be subject to
discipline based on having his clients prospectively waive malpractice li-
ability without having them represented by independent counsel in making
the waiver.

Module 5 Question 2

An attorney has just been admitted to practice law. The court would like
to appoint him to represent a defendant in a bigamy prosecution. The at-
torney believes that he is incompetent to handle the case because he
barely passed criminal procedure in law school and still does not under-
stand the topic. In addition, the attorney believes that bigamy should be
a capital offense. The attorney refuses the appointment after asking an-
other lawyer whether she would be interested in the appointment instead.

Is the attorney subject to discipline?

(A) Yes, because an attorney should not refuse a court appointment except
under very rare circumstances.

(B) Yes, because he did not try to associate himself with competent co-
counsel rather than refusing the appointment.
(C) No, because an attorney may refuse an appointment if he is incompetent
to handle the case or if his distaste for the case would impair his per-
formance.

(D) No, because the attorney attempted to engage substitute counsel before
he refused the court appointment.

The correct answer is: (C) No, because an attorney may refuse an appoint-
ment if he is incompetent to handle the case or if his distaste for the
case would impair his performance.

This answer choice is correct because it accurately reflects the excep-


tions to the general rule that attorneys should not refuse court appoint-
ments. An attorney can refuse an appointment for good cause, including the
likelihood that it will cause the lawyer to violate the Rules, such as in-
competently representing the client. He may also refuse when he finds the
client or the cause so repugnant that it is likely to impair the represen-
tation. In this case, the attorney's extreme view that bigamists should be
given the death sentence may affect his ability to represent the defend-
ant, as may his lack of experience. As a result, this answer choice offers
two plausible exceptions to the general rule that an attorney should not
refuse court appointments.

(A) Incorrect. Yes, because an attorney should not refuse a court appoint-
ment except under very rare circumstances.

This answer choice is incorrect because there are not-so-rare exceptions


to the general rule that an attorney should not refuse court appointments.
An attorney can refuse an appointment for good cause, including the like-
lihood that the representation will cause him to violate the Rules due to
impairment or incompetence to represent the client. He may also refuse
when he finds the client or the cause so repugnant that it is likely to
impair the representation, or when the representation is likely to impose
an unreasonable burden on the attorney.

(B) Incorrect. Yes, because he did not try to associate himself with com-
petent co-counsel rather than refusing the appointment.

This is not as complete as the correct answer. While the attorney could
have associated himself with competent co-counsel or otherwise improved
his competence to handle the appointment, he should still not have taken
the appointment because his extreme view that bigamists should be put to
death would most likely impair his ability to represent the client fairly.

(D) Incorrect. No, because the attorney attempted to engage substitute


counsel before he refused the court appointment.

The issue here is that the attorney had two reasons for not accepting the
appointment: his lack of competence and the fact that he is strongly bi-
ased against bigamists. The fact that he attempted to engage substitute
counsel is irrelevant.
Module 6 Question 1

A taxi driver was arrested for carrying a loaded gun in his cab, which is
a felony in this jurisdiction. The taxi driver hired a lawyer. Although
she had no idea on what basis to do so, the lawyer decided to file a mo-
tion to dismiss. She reserved a hearing date and time and notified the
prosecution of her intention to make the motion. She researched the rele-
vant statutes and state and federal case law, but found no legal authority
to support her motion. Then she reviewed the police report to see if there
was a factual basis to challenge the prosecution, but found none. The law-
yer decided to argue that the law inadvertently encouraged discrimination
against poor people because unprotected cab drivers are less likely to
drop off or pick up passengers in poor neighborhoods. She doubted her mo-
tion would succeed.

Should the lawyer be subject to discipline?

(A) Yes, because a lawyer shall not bring or defend a proceeding, or as-
sert or controvert an issue, unless she has a nonfrivolous basis in law
and fact for doing so.

(B) Yes, because a lawyer may not bring a motion for which her only argu-
ment is to reverse the law when the lawyer does not believe that the argu-
ment will succeed.

(C) No, because a lawyer for the defendant in a criminal proceeding may
defend the proceeding as to require that every element of the case be es-
tablished.

(D) No, because a lawyer is permitted to bring a motion for which her only
argument is that the law should be changed, even if the lawyer believes it
is unlikely to succeed.

The correct answer is: (D) No, because a lawyer is permitted to bring a
motion for which her only argument is that the law should be changed, even
if the lawyer believes it is unlikely to succeed.

The Model Rules state that a lawyer should not bring or defend a proceed-
ing or claim without a nonfrivolous basis in fact or law for so doing. The
nonfrivolous basis can include a good faith argument for an extension,
modification or reversal of existing law. Thus, this answer choice is cor-
rect. A lawyer may bring a motion for which no basis in law exists, no
facts in the record support the motion, and the lawyer's only argument is
to reverse the law. Nothing in the facts suggests that the lawyer did not
believe that she had a good-faith argument to reverse an existing law.
Moreover, the fact that the lawyer does not think the argument will suc-
ceed does not make the argument frivolous.

(A) Incorrect. Yes, because a lawyer shall not bring or defend a proceed-
ing, or assert or controvert an issue, unless she has a nonfrivolous basis
in law and fact for doing so.

This answer choice is not the best answer because it is an incomplete


statement of Model Rule 3.1, which prohibits frivolous motions. It is true
that a lawyer shall not bring or defend a proceeding, or assert or contro-
vert an issue therein unless there is a basis in law and fact for doing so
that is not frivolous. But MR 3.1 goes on to say that a motion is not
frivolous if it includes a "good-faith argument for reversal of an exist-
ing law. "

(B) Incorrect. Yes, because a lawyer may not bring a motion for which her
only argument is to reverse the law when the lawyer does not believe that
the argument will succeed.

This answer choice is not the best answer because a lawyer may bring a mo-
tion for which no basis in law exists, with no supporting facts in the
record, where the lawyer's only argument is to reverse the law, and that
the lawyer believes will fail. As long as the lawyer is making a good-
faith argument to reverse an existing law, the motion is not frivolous.

(C) Incorrect. No, because a lawyer for the defendant in a criminal pro-
ceeding may defend the proceeding as to require that every element of the
case be established.

This answer choice is incorrect because it is irrelevant. It is true that


a lawyer for the defendant in a criminal proceeding may so defend the pro-
ceeding as to require that every element of the case be established. But
this is a pretrial motion based only on the law, before the prosecution
attempts to establish its case.

Module 6 Question 2

A driver had standard liability coverage with an insurer. The policy pro-
vided that the insurer would provide a lawyer to defend the driver in
suits arising out of accidents, and it would pay for any settlement or
judgment against the driver. In addition, the driver would consent to the
insurer deciding whether to settle or try any suits.

The driver struck a pedestrian at an intersection, and a lawyer provided


by the insurer is representing the driver. The police report states that
the driver was not wearing glasses or contacts at the time. The driver had
claimed on his insurance application that he had 20/20 vision without
glasses or contacts. The insurer decides to investigate the issue because,
if the driver lied on the application, then the policy would be void and
the insurer would not be responsible.

The insurer informs the lawyer for the driver of the investigation and the
consequences to the driver for lying on the application. The insurer asks
the lawyer whether she noticed if the driver wore glasses or knew whether
the driver wore contacts. The insurer reminds the lawyer that she cannot
assist in a client's fraud. The lawyer feels sorry for the driver, an el-
derly man, and refuses to answer the insurer's questions.

Is the lawyer subject to discipline?


(A) Yes, because the lawyer owes a fiduciary duty to the insurer as a
third-party payor to provide the insurer with all information relating to
the representation.

(B) Yes, because the driver consented to have the insurer decide whether
to settle or try lawsuits, making the insurer the client to whom the law-
yer owes a duty of candor.

(C) No, because the lawyer cannot discuss any aspect of the driver's case
with the insurer, as the driver is the client and the insurer is only a
third-party payer to whom the lawyer owes no fiduciary duty.

(D) No, because the lawyer is not required to reveal information relating
to the representation of the driver, because the insurer is only a third-
party payor.

The correct answer is: (D) No, because the lawyer is not required to re-
veal information relating to the representation of the driver, because the
insurer is only a third-party payor.

The Model Rules require that, when a third party is paying for the repre-
sentation, the client must give his informed consent, the third party
payor must not interfere with the lawyer's professional judgment, and the
lawyer must preserve the client's confidentiality. Here, the insurer is
the third-party payor and the driver is the client. Therefore, the lawyer
may not relate information to the insurer regarding the representation of
the driver without the driver's consent.

The Rules do permit a lawyer to reveal confidential information to prevent


a client from using the lawyer's services to commit a fraud. The facts
here suggest that the insurer suspects a fraud, but there is no suggestion
that the lawyer's services are being used to further a fraud. Rather, the
insurer is attempting to use the lawyer to determine whether a fraud was
committed in the past. Therefore, this answer choice is correct.

(A) Incorrect. Yes, because the lawyer owes a fiduciary duty to the in-
surer as a third-party payor to provide the insurer with all information
relating to the representation.

This answer is incorrect because it misstates the Rule governing third-


party payors. Conflicts can arise where a third-party, such as an insur-
ance company or a parent, is paying for the legal representation of a cli-
ent. MR 1.8 requires that in situations where a third-party payor is in-
volved, the client must give his informed consent, the third-party payor
must not interfere with the lawyer's professional judgment, and the lawyer
must protect the client's confidentiality according to the Rules. There-
fore, information relating to the legal representation is confidential be-
tween the lawyer and the client unless the client consents otherwise.

(B) Incorrect. Yes, because the driver consented to have the insurer de-
cide whether to settle or try lawsuits, making the insurer the client to
whom the lawyer owes a duty of candor.
This answer is incorrect because when the driver consented that the in-
surer would decide whether or not to settle a lawsuit or go to trial, the
driver did not cease being the client. The insurer remains a third-party
payor, and the driver remains the client. The lawyer may not give confi-
dential information relating to the driver's legal representation to the
insurer without the driver's consent.

(C) Incorrect. No, because the lawyer cannot discuss any aspect of the
driver's case with the insurer, as the driver is the client and the in-
surer is only a third-party payer to whom the lawyer owes no fiduciary
duty.

This answer is overly broad and thus incorrect. Some information will be
related to the insurer so that it can make decisions about whether or not
to settle the case or go to trial. The driver consented to that much. How-
ever, confidential information unrelated to the insurer's decision to set-
tle or not must not be discussed by the lawyer and the insurer absent con-
sent on the part of the driver.

Module 6 Question 3

An attorney's 73-year-old client dies suddenly leaving his two daughters


as the only living relatives. The client's will leaves all of his consid-
erable estate to the young woman who walks his dog. The daughters have de-
cided to contest the will. At the trial, the attorney is contacted by the
press on his way into the courthouse. He states, "The daughters of my cli-
ent allege that his will was written under duress, but our answer details
several conversations that my client had with neighbors and friends in
which he firmly stated that he wanted to leave his estate to the dog
walker because she was such a sweet girl."

Has the attorney violated the Model Rules of Professional Conduct?

(A) No, because a lawyer may publicly state matters that are part of the
public record.

(B) No, because discovery had concluded.

(C) Yes, because trial publicity is not permitted regarding expected tes-
timony.

(D) Yes, because trial publicity is not permitted regarding the character
of a witness.

The correct answer is: (A) No, because a lawyer may publicly state matters
that are part of the public record.

This answer choice is correct, because the attorney was relating infor-
mation contained in the answer, which is a part of the public record. Rule
3.6 prevents trial publicity that is prejudicial to a proceeding. Relat-
ing matters that have become part of the public record, however, is ex-
pressly permitted by the Rule.
(B) Incorrect. No, because discovery had concluded.

This answer choice is incorrect, because the applicable Rule, 3.6, regard-
ing trial publicity makes no mention of this timing consideration. The
reason that the attorney has not violated the Rule is that he only spoke
of information contained in the public record.

(C) Incorrect. Yes, because trial publicity is not permitted regarding ex-
pected testimony.

Although trial publicity is not permitted regarding expected testimony,


the attorney in this case was relating information contained in the public
record. Such statements are expressly permitted by Rule 3.6. Thus, this
answer is incorrect.

(D) Incorrect. Yes, because trial publicity is not permitted regarding the
character of a witness.

This answer is incorrect, because even though the attorney's statement


makes mention of the deceased client's opinion about the dog walker's
character, the attorney is not asserting that the dog walker is in fact a
"sweet girl." The attorney is conveying information from the answer that
has become a part of the public record, and it is therefore permissible.

Module 7 Question 1

A seller and buyer are represented by separate counsel for the sale of real
property. During the course of negotiations, the buyer calls the seller's
lawyer to discuss some of the particulars of the real property. They
discuss the tax implications of the contract, and the lawyer provides
details of a group legal plan she could offer employees of the business
that the buyer intends to move to the subject property.

Is the lawyer subject to discipline for her communications with the buyer?

(A) Yes, because she has engaged in direct solicitation of professional


employment for her own pecuniary gain.

(B) Yes, because she has communicated with a represented party about the
matter that is the subject of the representation.

(C) No, because the buyer initiated contact with the lawyer.

(D) No, because the issues discussed were not related to a matter in an-
ticipation of litigation.

The correct answer is: (B) Yes, because she has communicated with a repre-
sented party about the matter that is the subject of the representation.

Under the Model Rules, a lawyer may not communicate about the subject of
the representation with a person the lawyer knows to be represented by an-
other lawyer in the matter, unless the lawyer (1) has the other lawyer's
consent, or (2) is authorized to do so by law or court order. The rule ap-
plies even where (as here) the represented person initiates the communica-
tion. The lawyer here has communicated with a represented party about the
subject of the representation (i.e., the land) without meeting either ex-
ception, and is therefore subject to discipline.

(A) Incorrect. Yes, because she has engaged in direct solicitation of pro-
fessional employment for her own pecuniary gain.

The Model Rules state that the rule against speaking to represented par-
ties about the subject of the representation does not prohibit a lawyer
from contacting "representatives of organizations or groups that may be
interested in establishing a group or prepaid legal plan for the purpose
of informing such entities of the availability of and details concerning
the plan or arrangement which the lawyer or lawyer's firm is willing to
offer." In other words, the prohibition against direct solicitation of le-
gal services does not apply to the facts in this case, and this answer
choice is incorrect.

(C) Incorrect. No, because the buyer initiated contact with the lawyer.

An attorney may not speak to a party that he or she knows is represented


by counsel regarding the subject of representation, regardless of whom in-
itiates the conversation. Thus, this answer is incorrect.

(D) Incorrect. No, because the issues discussed were not related to a mat-
ter in anticipation of litigation.

The Model Rules prohibit communications with represented parties about the
matter that is the subject of the representation regardless of whether or
not the matter is in anticipation of litigation. Thus, this answer is in-
correct.

Module 8 Question 1

Two individuals involved in a dispute agree to have an attorney serve as a


mediator to help resolve the issue. The attorney meets with the two unrep-
resented parties individually and together during the mediation, but he is
unable to mediate a settlement. During one of the individual meetings, one
of the parties commented that she was "glad to have the attorney represent
her during the mediation." At no time did the attorney tell the parties he
was acting solely as a neutral mediator or that he did not represent ei-
ther party in an individual capacity. During a deposition in a subsequent
trial on the matter, the attorney was subpoenaed and testified to state-
ments made to him privately by the parties during the mediation.

Is the attorney subject to discipline?

(A) No, because the attorney was acting in a non-attorney capacity and
thus not subject to disciplinary rules.

(B) No, because the attorney was not representing either party, and thus
no conflict of interest problems existed.

(C) Yes, because the attorney's actions violated the attorney-client priv-
ilege.

(D) Yes, because the attorney had a duty to inform the parties that he was
not representing them in an individual capacity.

The correct answer is: (D) Yes, because the attorney had a duty to inform
the parties that he was not representing them in an individual capacity.
The Model Rules state that a lawyer serving as a third-party neutral must
inform unrepresented parties that he or she is not, in fact, representing
them. In addition, when the lawyer knows (or should know) that a party
does not understand the lawyer's role in the matter, the lawyer must also
explain the difference between the lawyer's role as a third-party neutral
and a lawyer's role as one who represents a client. The attorney in the
present scenario, therefore, had a duty to inform the parties of his lim-
ited role from the outset, as well as to explain the differences between
his role and that of an attorney when it became apparent that one of the
parties did not understand this limited role.

(A) Incorrect. No, because the attorney was acting in a non-attorney ca-
pacity and thus not subject to disciplinary rules.

This answer is incorrect because it is contrary to the Rules. The Model


Rules, in fact, impose an obligation on attorneys acting in a non-attorney
capacity to inform unrepresented parties that he is not representing them,
and further to explain the differences between his role as mediator and
that of a lawyer representing a party when he "knows or reasonably should
know that a party does not understand the lawyer's role in the matter."

(B) Incorrect. No, because the attorney was not representing either party,
and thus no conflict of interest problems existed.

The attorney did not create a conflict of interest by serving as mediator


in this matter; he is nonetheless subject to discipline for failing to ex-
plain his role as mediator. Thus, this answer is incorrect.

(C) Incorrect. Yes, because the attorney's actions violated the attorney-
client privilege.

Because the attorney was not representing either party, he did not violate
the attorney-client privilege. Thus, this answer is incorrect.

Module 8 Question 2

A retired butcher purchased a small condominium from a general contractor.


Shortly after he moved in, he discovered numerous defects, such as mold
and water leaks, on the premises. After several months of unsuccessful at-
tempts to get the contractor to fix the problems, the butcher filed a pro
se suit. At the first status conference, the judge ordered the butcher and
the contractor's attorney to participate in the court's mediation service
before proceeding further. At the mediation, the state mediator asked the
butcher to explain his case to her first, outside the presence of the con-
tractor's counsel, then asked the contractor's counsel to do the same. At
a joint session following the private meetings, the contractor's attorney
told the butcher he would never be able to win and he should take a small
settlement. Then, in another private meeting, the mediator told the
butcher, "The contractor's attorney is right. You're a retired butcher
who's never been in a lawsuit. You could end up with nothing." The butcher
asked, "Do you really think that's best?" The mediator replied, "I do."
Based on her advice, the butcher settled the case for a nominal sum.

Is the mediator likely to be disciplined?

(A) No, because she met with the butcher privately, did not reveal to the
contractor's counsel what she told him, and held a joint session of all
parties to discuss settlement.
(B) No, because she simply reminded the butcher that he was a retired
butcher, explained to him that he could end up with nothing, and advised
him that she thought accepting the offer was best.

(C) Yes, because she failed to explain to the butcher that she didn't rep-
resent him and didn't tell him the difference between a mediator and a
lawyer representing a client.

(D) Yes, because she stepped out of the role of neutral third party and
gave the butcher explicit legal advice, and as a result, he settled the
case for what little was offered.

The correct answer is: (C) Yes, because she failed to explain to the
butcher that she didn't represent him and didn't tell him the difference
between a mediator and a lawyer representing a client.

Model Rule 2.4, Lawyer Serving as Third-Party Neutral, states: "(a) A law-
yer serves as a third-party neutral when the lawyer assists two or more
persons who are not clients of the lawyer to reach a resolution of a dis-
pute or other matter that has arisen between them." Comment [3] adds: "Un-
like nonlawyers who serve as third-party neutrals, lawyers serving in this
role may experience unique problems as a result of differences between the
role of a third-party neutral and a lawyer's service as a client repre-
sentative. The potential for confusion is significant when the parties are
unrepresented in the process. Thus, paragraph (b) requires a lawyer-neu-
tral to inform unrepresented parties that the lawyer is not representing
them. For some parties, particularly parties who frequently use dispute-
resolution processes, this information will be sufficient. For others,
particularly those who are using the process for the first time, more in-
formation will be required. Where appropriate, the lawyer should inform
unrepresented parties of the important differences between the lawyer's
role as third-party neutral and a lawyer's role as a client representa-
tive." Here, the mediator failed to explain to the butcher that she didn't
represent him and the difference between a mediator and a lawyer repre-
senting a client. The rule explicitly requires her to do so when she rea-
sonably should know that a party does not understand the lawyer's role in
the matter.

(A) Incorrect. No, because she met with the butcher privately, did not re-
veal to the contractor's counsel what she told him, and held a joint ses-
sion of all parties to discuss settlement.

This choice is incorrect because it doesn't address what the mediator did
wrong. It was not improper for her to meet with the butcher privately, nor
for her to reveal what she thought about the attorney being correct, nor
to hold a joint session to discuss settlement. The problem was that she
didn't warn the butcher, per the Model Rules, that she wasn't his lawyer
and that her opinion about settling was not the same as legal advice.
Thus, this answer is incorrect.

(B) Incorrect. No, because she simply reminded the butcher that he was a
retired butcher, explained to him that he could end up with nothing, and
advised him that she thought accepting the offer was best.

It was not improper for the mediator to remind the instructor he was a re-
tired butcher, explain to him that he could end up with nothing, and ad-
vise him that she thought accepting the offer was best. The problem was
that she didn't warn him that she wasn't his lawyer and that her opinion
about settling was not the same as legal advice. Thus, this answer is in-
correct.
(D) Incorrect. Yes, because she stepped out of the role of neutral third
party and gave the butcher explicit legal advice, and as a result, he set-
tled the case for what little was offered.

Other than her failure to warn the butcher as the rules require that she
was not his attorney and that her opinion did not constitute legal advice,
the mediator did not violate her duty as a neutral third party. Indeed,
advising the butcher of her opinion is, in part, what neutrals are sup-
posed to do. The butcher's reliance on the mediator's advice may very well
have been the best thing for him to do and, in any case, does not suggest
that the mediator should be disciplined. Thus, this answer is incorrect.

Module 9 Question 1

A lawyer agrees to represent a personal injury plaintiff on a contingent


fee basis. The fee agreement specifies that the attorney will be entitled
to 30% of any recovery. The lawyer obtains a $200,000 verdict. The attor-
ney advanced $50,000 in costs over the course of the litigation. The at-
torney deposits all $200,000 into an appropriate trust account and con-
tacts the client immediately regarding disbursement of the money. The
lawyer advises the client that he will take $60,000 in fees (30% of
$200,000) and $50,000 in reimbursement for costs advanced. As a result,
the client will receive a check for $90,000. The client argues that the
attorney should take his 30% fee from the amount of the verdict after ex-
penses have been deducted. The client does not dispute that the attorney
is entitled to reimbursement of the $50,000, but he argues that the law-
yer's fee should be $45,000 (30% of the remaining $150,000) and that the
client is entitled to $105,000. The fee agreement is silent as to whether
the attorney's fee should be computed based on the amount obtained before
or after costs are deducted.

How much money must the attorney keep in the trust account while the at-
torney and the client resolve their dispute?

(A) $200,000.

(B) $150,000.

(C) $105,000.

(D) $15,000.

The correct answer is: (D) $15,000.

Under the Model Rules, when there are two people who claim an interest in
funds held in a trust account, the lawyer is obligated to keep the dis-
puted amount in a trust account until the dispute is resolved. [MR 1.15.]
In this case, the client claims he is entitled to $105,000 while the at-
torney maintains that the client is only entitled to $90,000. Therefore,
there is only $15,000 in dispute in this case. The attorney may keep
$50,000 for reimbursement of costs advanced, and $45,000 in legal fees.
Moreover, the attorney must disburse $90,000 to the client, as the parties
agree that the client is entitled to at least $90,000. The remaining
$15,000 must remain in the trust account until the dispute is resolved.

(A) Incorrect. $200,000.

It is not necessary for the lawyer to keep the entire $200,000 in the
trust account (nor is the lawyer allowed to do so). Rather, only the
amount in dispute may be kept in the account, and the lawyer must promptly
deliver to the client any funds from the trust to which the client is en-
titled. In this case, only $15,000 is actually in dispute (the difference
between the $105,000 that the client claims an interest in and the $90,000
that he lawyer claims is the client's share.) Therefore, the lawyer must
keep $15,000 in the trust account while the parties resolve their dispute.

(B) Incorrect. $150,000.

This answer choice is tempting because it seems as though the only thing
the lawyer and the client agree upon is that the lawyer is entitled to re-
imbursement of the costs advanced, which is $50,000. However, in this
case, the client claims he is entitled to $105,000 while the attorney
maintains that the client is only entitled to $90,000. Therefore, there
is only $15,000 in dispute in this case. The client is entitled to receive
$90,000, while the attorney is entitled to take $95,000 from the account
($50,000 for advanced costs and $45,000 in legal fees which are not in
dispute). The lawyer must leave the remaining $15,000 in the trust ac-
count until the dispute is resolved.

(C) Incorrect. $105,000.

Under the Model Rules, when there are two people who claim an interest in
funds held in a trust account, the lawyer is obligated to keep the dis-
puted amount of money in the trust account until the dispute is resolved.
[MR 1.15.] However, the lawyer may only keep the disputed amount in the
trust account. He must promptly deliver to the client any funds from the
trust to which the client is entitled. In this case, the client claims he
is entitled to $105,000 while the attorney maintains that the client is
only entitled to $90,000. However, the entire amount is not in dispute.
Both parties agree that he client is entitled to at least $90,000, so the
lawyer must disburse $90,000 to the client right away. Moreover, they
agree that the lawyer is entitled to $50,000 for costs advanced and at
least $45,000 in fees. Therefore, only the remaining $15,000 is in dis-
pute, and only this amount must remain in the trust account pending reso-
lution of the parties' dispute.

Module 9 Question 2

A lawyer represented a client in complex and lengthy litigation. The law-


yer received a $4 million check in payment of the judgment the lawyer ob-
tained for the client, which the lawyer deposited in his client trust ac-
count. The lawyer and the client then had a dispute over how much money
the lawyer is owed in fees and costs.

Which of the following actions is neither recommended nor required by the


Model Rules?

(A) Paying the client the amount that is not in dispute.

(B) Suggesting a means for prompt resolution of the dispute.

(C) Withdrawing from the client trust account the amount of fees and costs
that are not in dispute.

(D) Notifying the appropriate State Bar of the dispute.


The correct answer is: (D) Notifying the appropriate State Bar of the dis-
pute.

Under the Model Rules, when there is a dispute over the amount of fees and
costs owed to an attorney out of funds received on behalf of a client, the
attorney must promptly distribute the undisputed portion of the funds. The
attorney should also suggest to the client an appropriate means for re-
solving the dispute, such as arbitration. However, the rules neither re-
quire nor recommend that the attorney report the dispute to the State Bar.

(A) Incorrect. Paying the client the amount that is not in dispute.

This answer is contrary to the Model Rules and thus incorrect. In fact,
the Model Rules state that where there is a dispute over the amount of
fees and costs owed to an attorney out of funds received on behalf of a
client, the attorney must promptly distribute the undisputed portion of
the funds. Thus, this answer is incorrect.

(B) Incorrect. Suggesting a means for prompt resolution of the dispute.

Under the Model Rules, an attorney should also suggest to a client an ap-
propriate means for resolving a fee dispute, such as arbitration. Thus,
this answer is incorrect.

(C) Incorrect. Withdrawing from the client trust account the amount of
fees and costs that are not in dispute.

According to the Model Rules, where there is a dispute over the amount of
fees and costs owing to an attorney out of funds received on behalf of a
client, the attorney must promptly distribute the undisputed portion of
the funds. Thus, this answer is incorrect.

Module 10 Question 1

A lawyer who specializes in divorce and custody law realizes that there
are very few family lawyers on the east side of town. She also notices a
newspaper article in the local paper describing the high rate of divorce
in communities on the east side of town. In an effort to reach as many
potential clients as possible, she researches court records to locate pro
se divorce filings and prepares an advertising circular containing
information about her office and her practice to mail to all unrepresented
divorce litigants. She mails the circular to them in an envelope with her
name and return address.

Did the attorney act properly?

(A) Yes, because a lawyer may advertise to a pro se litigant as a


service to the litigant.

(B) Yes, because mailings cannot be targeted to those known to be in need


of a particular legal service.

(C) No, because the mailings targeted those known to be in need of a


particular legal service.

(D) No, because she must wait until 30 days after a pro se litigant files
before contacting him or her.
The correct answer is: (A) Yes, because a lawyer may advertise to a pro se
litigant as a service to the litigant.

Because this is advertising rather than in-person solicitation (or


electronic real-time solicitation), the lawyer may actually send it to
anyone, regardless of their representation status. In addition, the
advertisement must include the name and contact information of the
lawyer responsible for its content, which it does in this case [MR
7.2(d)].

(B) Incorrect. Yes, because mailings cannot be targeted to those known


to be in need of a particular legal service.

The Model Rules do contain provisions about sending legal advertising


circulars. Specifically, a lawyer may send circulars targeted to those
known to be in need of particular legal services if they meet the
general requirements for advertising.

(C) Incorrect. No, because the mailings targeted those known to be in


need of a particular legal service.

The Model Rules do contain provisions about sending legal advertising


circulars. Specifically, a lawyer may send circulars targeted to those
known to be in need of particular legal services if they meet the
general requirements for advertising.

(D) Incorrect. No, because she must wait until 30 days after a pro se
litigant files before contacting him or her.

Some states have rules that lawyers cannot send targeted direct mailings
to accident or disaster victims and their families for 30 days after the
disaster or accident. However, the Model Rules do not contain this
limitation or any other time limitation as to targeted advertising.

Module 10 Question 2

While driving home, a lawyer sees a three-car collision. There are no po-
lice or ambulances at the scene and it appears the accident has just oc-
curred. The lawyer, who is also a licensed medical doctor, pulls her car
to the side of the road, calls 911 on her cell phone and then grabs her
first aid kit and tries to assist the people involved in the accident.
While bandaging a bleeding woman, the clearly traumatized woman exclaims
repeatedly, "That jerk! He was talking on his cell phone and ran a red
light. What am I going to do? I can't afford this!" The lawyer tells the
woman, "I'm a doctor but I'm also a lawyer. I just wanted to let you know
that you should obtain legal representation as soon as possible. You can
call me if you have any questions, free of charge." The lawyer then gives
the woman her card.

Was the lawyer's conduct proper?

(A) Yes, because the woman was clearly in need of legal services and asked
for advice.

(B) Yes, because attorneys are permitted to advise people of the need to
obtain legal counsel.
(C) No, because in-person solicitation of a specific prospective client is
completely prohibited under the Model Rules.

(D) No, because the woman was clearly traumatized and under duress.

The correct answer is: (B) Yes, because attorneys are permitted to advise
people of the need to obtain legal counsel.

The Model Rules prohibit in-person solicitation of employment from a pro-


spective client when the attorney's significant motivation is pecuniary
gain. [MR 7.3] A lawyer also cannot engage in solicitation that involves
coercion, duress or harassment. Thus, it is generally not permissible for
a lawyer who witnesses an accident to hand out her card to victims at the
scene and offer her legal services. But there are certain exceptions. For
instance, a lawyer does not violate the Model Rules by explaining the
need for legal services and offering legal services without charge or
suggesting the person seek legal help. In this case, it was permissible
for the lawyer to advise the injured woman of her need for legal services
and to offer to answer her questions free of charge. The fact that the
lawyer initially stopped to offer medical assistance and offered legal
assistance free of charge dispels any concern that her motivation may be
pecuniary gain. Therefore, this answer is correct.

(A) Incorrect. Yes, because the woman was clearly in need of legal ser-
vices and asked for advice.

Generally, a lawyer cannot solicit fee-generating employment by in-person,


live telephone, or real-time electronic contact. [MR 7.3] While the in-
jured woman in this case may have clearly needed legal services, that fact
does not justify in-person solicitation. The fact that the woman asked
"What am I going to do?" also does not excuse in-person solicitation be-
cause the woman was not specifically asking for legal advice or seeking a
lawyer. What does save the lawyer's in-person solicitation is that it was
made without financial motivation.

(C) Incorrect. No, because in-person solicitation of a specific prospec-


tive client is completely prohibited under the Model Rules.

The Model Rules generally prohibit in-person solicitation of fee-generat-


ing employment from a specific prospective client. [MR 7.3] However, there
are certain exceptions to that general rule. For example, a lawyer can so-
licit another lawyer or someone with whom the lawyer has a family, close
personal, or prior professional relationship. In this instance, the lawyer
did not violate the Model Rules because she was not seeking fee-generating
employment. Therefore, it is incorrect to state that in-person solicita-
tion is completely prohibited under all circumstances.

(D) Incorrect. No, because the woman was clearly traumatized and under du-
ress.

Model Rule 7.3 generally prohibits in-person solicitation of prospective


clients when a significant motivation for the solicitation is the lawyer's
financial gain. A lawyer also cannot engage in solicitation involving du-
ress, coercion or harassment. The woman in this accident scenario could be
considered under duress. However, that does not automatically render the
lawyer's conduct improper. The lawyer offered assistance free of charge
and thus had no motivation for significant pecuniary gain. The Model Rules
do not prohibit an attorney from alerting people to the fact that they
need legal counsel or offering services free of charge. Therefore, this
answer is incorrect
Module 11 Question 1

A district attorney asked a grand jury to indict a contractor for bribery


and racketeering. The district attorney does not have probable cause to
believe that the contractor actually committed the crimes, and does not
intend to prosecute the contractor, but he wants to use the indictment as
leverage to try to get him to testify against his friends, who are the
leaders of a major drug-trafficking organization.

Should the district attorney be disciplined for this conduct?

(A) No, because inducing the contractor to testify would be "in the inter-
est of justice."

(B) No, because he does not intend to prosecute the contractor.

(C) Yes, because attorneys may not present false statements or evidence
before a tribunal.

(D) Yes, because he doesn't have sufficient evidence to support an indict-


ment.

The correct answer is: (D) Yes, because he doesn't have sufficient evi-
dence to support an indictment.

Model Rule 3.8 addresses the special duties of prosecutors. A prosecutor


in a criminal case shall refrain from prosecuting a charge that the prose-
cutor knows is not supported by probable cause. The district attorney's
actions are inappropriate, regardless of his motivation. Even if getting
evidence against the contractor's friends is in the interest of justice
and the district attorney doesn't intend to actually prosecute the con-
tractor, his behavior is sanctionable because he does not have sufficient
evidence to support an indictment against him.

(A) Incorrect. No, because inducing the contractor to testify would be "in
the interest of justice."

The standard for seeking an indictment is not "in the interest of jus-
tice." Rather, under the Model Rules, a prosecutor in a criminal case
shall refrain from prosecuting a charge that the prosecutor knows is not
supported by probable cause. Thus, this answer is incorrect.

(B) Incorrect. No, because he does not intend to prosecute the contractor.

Whether or not the district attorney actually prosecutes the contractor is


irrelevant. According to the Model Rules, the prosecutor in a criminal
case must refrain from prosecuting a charge that the prosecutor knows is
not supported by probable cause. His behavior is sanctionable because he
does not have sufficient evidence to support an indictment against the
contractor.

(C) Incorrect. Yes, because attorneys may not present false statements or
evidence before a tribunal.

Model Rule 3.3 provides that an attorney may not make a false statement of
fact or law to a tribunal, or offer evidence that the attorney knows to be
false. However, the facts do not indicate that the attorney is prepared to
make a false statement or present false evidence. All of the evidence and
testimony presented by the prosecutor could be entirely truthful, even
though the case was brought without probable cause. Thus, this answer is
incorrect.

Module 11 Question 2

A community suffered several home invasions in which the perpetrator tied


up the occupants and ransacked the homes for cash and jewelry. The entire
city was on edge until the perpetrator was caught outside a home that had
just been ransacked. Several eyewitnesses also identified him in a police
line-up as the person they had seen leaving other homes that had been in-
vaded. The prosecutor told the media at a press conference: "Due to dili-
gent efforts of our city's finest, we have the perpetrator of these des-
picable acts. We congratulate the police on this arrest. These crimes ter-
rorized our city, and we will prosecute this guilty perpetrator to the
fullest extent of the law."

The suspect was held without bail. During the course of the prosecutor's
pretrial preparations, he learned that the suspect is the boyfriend of a
teenage girl who lives in the last home invaded. Furthermore, the suspect
had a solid alibi for the times at which two other home invasions took
place. The prosecutor duly passed all this information along to the de-
fense attorney. After more investigation, the prosecutor dropped the
charges.

Is the prosecutor subject to discipline?

(A) No, because the prosecutor promptly provided exculpatory information


to defense counsel.

(B) No, because the prosecutor dismissed the charges when he determined
that evidence was lacking.

(C) Yes, because the prosecutor's comments to the media were prohibited.

(D) Yes, because the prosecutor initiated a prosecution without probable


cause.

The correct answer is: (C) Yes, because the prosecutor's comments to the
media were prohibited.

Model Rule 3.6 provides that a lawyer who is participating or who has par-
ticipated in the investigation or litigation of a matter shall not make an
extrajudicial statement that the lawyer knows or reasonably should know
will be disseminated by means of public communication and will have a sub-
stantial likelihood of materially prejudicing an adjucative proceeding in
the matter. Moreover, Model Rule 3.8 provides that, except for statements
that are necessary to inform the public of the nature and extent of the
prosecutor's action and that serve a legitimate law enforcement purpose,
prosecutors must refrain from making extrajudicial comments that have a
substantial likelihood of heightening public condemnation of the accused.
Here, the prosecutor made statements to the press that would tend to in-
flame public opinion against the accused and had the potential to bias the
jury pool. Notwithstanding the fact that the prosecutor dropped the
charges, he could still be subject to discipline since his statement that
the individual was the perpetrator and was guilty could heighten public
condemnation of that individual. Thus, this is the best answer to this
question.

(A) Incorrect. No, because the prosecutor promptly provided exculpatory


information to defense counsel.

This is a correct statement, but it is not the best answer to this ques-
tion. The prosecutor did fulfill his obligation under Model Rule 3.8(d) to
"make timely disclosure to the defense of all evidence or information
known to the prosecutor that tends to negate the guilt of the accused or
mitigates the offense." A prosecutor who fails to promptly notify defense
counsel of evidence that tends to negate the defendant's guilt would be
subject to discipline. Here, the prosecutor did disclose favorable infor-
mation to defense counsel, so the prosecutor would not be subject to dis-
cipline for violating this rule. However, the prosecutor still made extra-
judicial statements in contravention of the Model Rules. Thus, the prose-
cutor will still be subjected to professional discipline.

(B) Incorrect. No, because the prosecutor dismissed the charges when he
determined that evidence was lacking.

This is not the best answer to this question, although it is true that the
prosecutor acted properly in terminating the prosecution. Prosecutors have
a special duty to proceed in a criminal prosecution only when the charge
is supported by probable cause. Here, the prosecutor properly terminated
the prosecution when it became clear that credible evidence linking the
defendant to the crimes charged was lacking. Notwithstanding this appro-
priate course of action, the prosecutor previously made extrajudicial
statements that subjected the accused to heightened public scrutiny. It is
irrelevant that the charges were subsequently dropped. Thus, this answer
choice is incorrect.

(D) Incorrect. Yes, because the prosecutor initiated a prosecution without


probable cause.

This answer is incorrect. On these facts, there was ample probable cause
to initiate the prosecution. The suspect was caught leaving a home that
had just been invaded, and several victims identified the suspect in po-
lice line-ups. The fact that evidence acquired later tended to exculpate
the suspect does not make the initial decision to prosecute improper.
Thus, this answer is incorrect.

Module 12 Question 1

A clerk has been clerking for a judge for the past three years. In the
past, the judge and the clerk have not always seen eye to eye on both per-
sonal and legal issues. When a position opens with another judge in the
court, the clerk decides she would like to take the position. The clerk
asks the judge for whom she currently clerks for a letter of recommenda-
tion and the judge agrees, knowing her recommendation will hold clout with
the other judge. The judge is also excited at the prospect of getting a
new clerk for herself. She writes a letter of recommendation for the
clerk on her official letterhead.

Will the judge be subject to discipline for writing the letter of recom-
mendation for her clerk?

(A) Yes, because the judge may not use her name, influence, or the pres-
tige of her office for this purpose.

(B) Yes, because the judge knew that the recommendation on her letterhead
would hold clout with the other judge.

(C) No, because the only requirement for a judge to use her letterhead for
a recommendation is that the recommendation is based on the judge's per-
sonal knowledge.

(D) No, because a judge may provide a reference such as the one in this
case on her official letterhead.

The correct answer is: (D) No, because a judge may provide a reference
such as the one in this case on her official letterhead.

In general, a judge may not use her name, influence, or the prestige of
her office to support private or personal causes. A judge also should not
allow others to think that the judge is in a "special position" to influ-
ence decisions that judges will make. However, a judge is permitted pro-
vide references or recommendations for individuals on official letterhead
if they are based on the judge's personal knowledge and won't be seen as
an attempt to exert pressure by reason of the judicial office. CJC Rule
1.3. In this case, the judge does have personal knowledge of the clerk's
qualification, as the clerk worked for the judge herself. In addition, it
is common for a judge to provide a recommendation for a law clerk looking
for a new job, and will most likely not be seen as an attempt to exert
pressure by reason of the judicial office. The fact that the judge knows
her letter will have "clout" does not mean that the judge is exerting
pressure through her letter; rather, this simply means that the judge
knows her letter will be influential (which is in fact the purpose of a
recommendation letter). Thus, this answer choice is correct.

(A) Incorrect. Yes, because the judge may not use her name, influence, or
the prestige of her office for this purpose.

It is true that a judge may not use her name, influence, or the prestige
of her office to support private or personal causes. However, a recommen-
dation letter for an employee would not be considered a private or per-
sonal cause. A judge may give references or recommendations on official
letterhead if they are based on the judge's personal knowledge and won't
be seen as an attempt to exert pressure by reason of the judicial office.
As such, this answer choice is incorrect.

(B) Incorrect. Yes, because the judge knew that the recommendation on her
letterhead would hold clout with the other judge.
A judge may not use her letterhead to provide a letter of recommendation
if it will be seen as an attempt to exert pressure by reason of the judi-
cial office. However, the facts in this case merely indicate that the
judge knew her letter would hold clout in other words, that the letter
would be influential. There is nothing to indicate that the judge is using
the letter as a vehicle to exert pressure beyond the level of any other
job recommendation. Therefore, this answer choice is incorrect.

(C) Incorrect. No, because the only requirement for a judge to use her
letterhead for a recommendation is that the recommendation is based on the
judge's personal knowledge.

A judge may give references or recommendations for individuals on official


letterhead if they are based on the judge's personal knowledge and won't
be seen as an attempt to exert pressure by reason of the judicial office.
In this case, the judge does have personal knowledge of the clerk's quali-
fication, as the clerk worked for the judge herself. In addition, it is
common for a judge to provide a recommendation for a law clerk looking for
a new job, so it would not be seen as an attempt to exert pressure by rea-
son of the judicial office. The fact that the judge knows her letter will
have "clout" does not mean that the judge is exerting pressure through her
letter. Rather, this simply means that the judge knows her letter will be
influential (which is usually the purpose of a recommendation letter).

Module 12 Question 2

A judicial clerk is acting as the judge's primary research clerk for a


complex litigation. A partner for one of the law firms involved is very
impressed by the clerk's work and wants to hire her as an associate to
work on cases other than the litigation before the judge. The clerk noti-
fies the judge and then proceeds to negotiate with the partner over her
terms of employment as an associate with the partner's firm.

Is the clerk's conduct proper?

(A) Yes, because she notified the judge before negotiating employment with
the firm.

(B) Yes, because a law clerk is not bound by the restrictions precluding
judges from negotiating for employment with lawyers involved in matters in
which the judge is participating personally and substantially.

(C) No, because a law clerk may not negotiate for employment with a party
or lawyer who is involved in a matter in which the law clerk is partici-
pating personally and substantially.

(D) No, because the partner and the partner's law firm did not withdraw
from representation in the complex litigation before the judge.

The correct answer is: (A) Yes, because she notified the judge before ne-
gotiating employment with the firm.
Under Model Rule 1.12(b), a lawyer serving as a judicial clerk can negoti-
ate for employment with a party or lawyer involved in a matter in which
the clerk is participating personally and substantially only after the
clerk has so notified the judge. Therefore, the clerk's conduct is proper,
and this is the correct answer.

(B) Incorrect. Yes, because a law clerk is not bound by the restrictions
precluding judges from negotiating for employment with lawyers involved in
matters in which the judge is participating personally and substantially.

This is a true statement of the law. However, the issue here is that a
lawyer serving as a law clerk may only negotiate for employment with a
party or lawyer in a matter in which the law clerk is personally and sub-
stantially participating after the law clerk has notified the judge or ad-
judicative officer who employs her. Thus, this is not the best answer to
this question.

(C) Incorrect. No, because a law clerk may not negotiate for employment
with a party or lawyer who is involved in a matter in which the law clerk
is participating personally and substantially.

This answer choice is an incorrect statement of the law. As stated in


Model Rule 1.12(b), a law clerk may in fact negotiate for employment with
a person involved as a party or lawyer in a matter in which the law clerk
is personally and substantially participating after the law clerk has no-
tified the judge or adjudicative officer who employs her. This rule is
different than that applicable to a lawyer who is participating "person-
ally and substantially" in a matter as an actual judge or other adjudica-
tive officer, or as a third-party neutral. Model Rule 1.12(b) precludes a
lawyer in that position from negotiating for employment with any person
involved in the matter as a party or lawyer.

(D) Incorrect. No, because the partner and the partner's law firm did not
withdraw from representation in the complex litigation before the judge.

It is true that a judge or law clerk cannot represent anyone in connection


with a matter in which the judge or law clerk participated personally and
substantially as a judge or law clerk, unless all parties to the proceed-
ing give informed written consent [MR 1.12(a)]. However, because the part-
ner here offered the law clerk work on other matters, neither consent nor
withdrawal is necessary.

You might also like